• 「数学」简单文化课积累本云端版


    whk 过程中遇到了一些自认为还挺有意思的题,就放在这里,毕竟我也没啥可以写 blog 的东西了(也许是怕纸质版丢了

    不过对于大家来说应该都是 sb 题。

    Upd on 2021/11/05: 由于 [数据删除],可能会加入部分 ( m MO) 甚至涉及部分微积分的题目。

    Problem #1

    ( riangle m ABC) 中,求 (cos Acos Bcos C) 的最大值。


    Analysis

    多元最大值问题考虑主元法,利用 (cos) 函数的最大值为 (1), 以及三角形内角和为 (pi) 的这两个条件进行解答。

    化简过程中应熟练运用积化和差 & 诱导公式 & 均值不等式这些东西,总之很简单就不大想多说了。

    Sol

    [egin{aligned} cos Acos Bcos C &= frac 12cos Aleft[ cos(B-C) + cos(B+C) ight] \ &= frac 12cos Aleft[ cos(B-C) - cos A ight] \ &leq frac 12cos Aleft( 1-cos A ight) \ &leq frac 12left[ frac {cos A + (1 - cos A)}{4} ight] = frac 18 end{aligned} ]

    等号 (mathbf{iff.} cos A = cos B = cos C = frac 12) 时取到。

    Problem #2

    已知 (a = 3ln 2^{pi}, b = 2ln 3^{pi}, c = ln pi^{6}), 求 (a,b,c) 的大小关系。


    Analysis

    Problem #3

    求函数 (f(x) = frac {cos x - 2}{sqrt{3 - 2cos x+sin x}}) 的值域。


    Analysis

    分离参数题里面稍微需要点代数技巧的题,总之就是把分子变成常数,发现未知部分是一个 (sin / cos) 的形式,那么就辅助角公式一算就行了。

    当然还是要用到一些单调性,同角弦三角函数平方和一些简单性质,难度也不是太大就不说了。

    Sol

    [egin{aligned} f(x) &= frac {cos x - 2}{sqrt{3 - 2cos x + sin x}} = - frac {2 - cos x}{sqrt{frac 12(6 - 4cos x + 2sin x)}} \ &= - sqrt {frac {(2 - cos x)^2}{frac 12[(4 - 4cos x + cos^2 x) + (1 + 2sin x + sin^2 x)]}} \ &= - sqrt {frac {2(2 - cos x)^2}{(2 - cos x)^2 + (1 + sin x)^2}} = - sqrt 2left[1 + left( frac {1 + sin x}{2 - cos x} ight)^2 ight]^{-frac 12} end{aligned} ]

    (u = frac {1 + sin x}{2 - cos x} ; (u geq 0)), 那么 (1 + sin x = 2u - ucos x), 即 (sin x + ucos x = 2u - 1).

    用辅助角公式,得到 (sqrt{u^2 + 1} sin (x + varphi) = 2u - 1), 即 (sin (x + varphi) = frac {2u - 1}{sqrt{u^2 + 1}}).

    (lvert frac {2u-1}{sqrt{u^2 + 1}} vert leq 1), 解得 (u in [0, frac 43]), 又 (-sqrt{2}(1+u^2)^{-frac 12})([0, frac 43]) 单调递增,所以 (f(x)) 值域为 ([-sqrt 2, -frac 35 sqrt 2]).

    Problem 4

    (f(x)) 是定义在 (Bbb R) 上且周期为 (1) 的函数,在区间 ([0, 1)) 上, (f(x) = egin{cases} x^2 & x in D \ x & x ot in D end{cases}), 其中 (D = { x | x = frac {n - 1}{n}, n in {Bbb N}^{*}}), 求方程 (f(x) = lg x) 的解的个数。


    Analysis

    这题是 (2017) 江苏的 ( m T_{14}), 有一说一确实比较神仙,但估计有点底子的 ( m MO) 同学一眼就能看出 (lg x) 在研究点是不存在有理点的,只需要算一下 (x otin D) 的情况就可了。

    主要我也不知道该咋说,就看出来只需要研究 ([1, 10)), 然后分析下去发现 (x in D) 根本没法有交点,最后画个草图一看就完了。

    Sol

    因为 (f(x) in [0, 1)), 所以只需要研究 (x in [1, 10)) 时根的所有情况。

    (D_i = {x | x = frac {n - 1}{n} + i, n in {Bbb N}^{*}, i in {Bbb N}^{*} cap [1,9]}), 显然 (forall i, D_i subseteq {Bbb Q}).

    那么根据题意,当 (x in [i, i + 1) ;i in {Bbb N}^{*} cap [1,9]) 时,(f(x) = egin{cases} (x-i)^2 & x in D_i \ x-i & x ot in D_i end{cases})

    (x in (1, 10) cap {Bbb Q}), 设 (x = frac qp, p,q in {Bbb N}^{*}, p geq 2, (p, q) = 1).

    (lg x in {Bbb Q}), 则因为 (lg x in (0, 1)), 设 (lg x = frac nm, n, m in {Bbb N}^{*}, m geq 2, (n, m) = 1).

    所以 (10^{frac nm} = frac qp), 则 (10^n = left( frac qp ight)^m), 此时 (10^n in {Bbb Z}, left( frac qp ight)^m otin {Bbb Z}), 矛盾,所以 (lg x otin {Bbb Q}).

    所以 (lg x) 不可能与 (x in D_i) 内的部分对应相等,只需考虑 (lg x) 与每个周期内 (x otin D_i) 部分的交点。

    ((lg x)' vert_{x=1} = frac {1}{ln 10} < 1), 所以在 ([1, 2)) 上只有一个交点,所以有八个交点,原方程有八个解。

    Problem 5

    解不等式:

    [x^{12}+3x^{10}+5x^8+3x^6+1<2(x^4+1) ]


    Analysis

    发现全是偶次幂,那么换元。然后会发现有个各项系数出现了 (1;3;5;3) 这个顺序,那么考虑立方和公式,最后会发现是个同构。

    虽然好像观察系数发现两边同除立方和公式构造同构这件事并不「显然」,但是代数化简就是这个样子((((

    Solution

    做代换 (t = x^2),则原式等价于 (t^6+3t^5+5t^4+3t^3<2t^2+1).

    两边同时除以 (t^3 = (x^2)^3 ge 0) 得到 (t^3+3t^2+5t+3<dfrac 2t+dfrac{1}{t^3}).

    对左边用立方和公式,写出同构 ((t+1)^3+2(t+1)<left(dfrac 1t ight)^3 + 2 cdot dfrac 1t).

    构造函数 (f(x)=x^3+2x),显然 (f(x))(R) 上单调递增,则 (f(t+1)<fleft(dfrac 1t ight)) 成立当且仅当 (t+1<dfrac 1t).

    解得 (0 < t le dfrac{sqrt{5}-1}{2}),那么 (0 le x le sqrt{dfrac 12(sqrt 5-1)})

    Problem 6

    已知 (n in N^*)(n ge 2),求证:

    [dfrac 47 < 1 - dfrac 12 + dfrac 13 - dfrac 14 + cdots + dfrac {1}{2n-1} - dfrac {1}{2n} < dfrac{sqrt 2}{2} ]


    Analysis

    重写数列不等式可以发现实际上是调和级数的一部分,然后左边就 ( m Titu's ;Lemma) 搞出来一个递增的下界,右边就 ( m Cauchy) 不等式.

    然后你会惊喜的发现,待证不等式那叫一个宽啊,建议左半边改成 (dfrac {7}{12}),读者可以自行尝试一下(((

    Solution

    易证:

    [sum_{i=1}^{2n} (-1)^{i-1} dfrac 1i = sum_{i=1}^{2n} dfrac 1i - 2left( sum_{j=1}^n dfrac{1}{2j} ight) = sum_{i=n+1}^{2n} dfrac 1i ]

    考虑 ( m Titu's ; Lemma)

    [sum_{i=n+1}^{2n} dfrac {(sqrt 1)^2}{i} geq dfrac{n^2}{(n+1)+(n+2)+cdots+2n} = dfrac{n^2}{frac 32n^2+frac 12n} = dfrac{2}{3+frac{1}{n}} ]

    显然该式关于 (n) 单调递增,取 (n=2) 得原不等式左边成立,考虑利用 ( m Cauchy-Schwarz) 不等式

    [egin{aligned} sum_{i=n+1}^{2n} dfrac{1}{i} cdot 1 &leq left[n imes left(dfrac{1}{(n+1)^2}+dfrac{1}{(n+2)^2}+cdots+dfrac{1}{(2n)^2} ight) ight]^{frac 12} \ &< sqrt nleft[ dfrac 1{n(n+1)} + dfrac 1{(n+1)(n+2)} + cdots + dfrac 1{(2n-1)(2n)} ight]^{frac 12} \ &= sqrt nleft[ dfrac 1n - dfrac {1}{n+1} + dfrac {1}{n+1} - dfrac 1{n+2} + cdots + dfrac {1}{2n-1} - dfrac {1}{2n} ight]^{frac 12} \ &= sqrt{n cdot left(dfrac 1n - dfrac 1{2n} ight)} = dfrac {sqrt 2}{2} end{aligned} ]

    Problem 7

    (x,y,z > 0),求下式的最大值.

    [f = dfrac{(2x+y+z)^2}{2x^2+(y+z)^2} + dfrac{(x+2y+z)^2}{2y^2+(x+z)^2} + dfrac{(x+y+2z)^2}{2z^2+(x+y)^2} ]


    Analysis

    经典轮换对称式最值,发现原式齐次,那么就做 (x+y+z=1) 假设,一通化简就完事了((((这题好简单啊为什么要放上来啊

    Solution

    (x,y,z) 关于 (f) 的齐次性(即用 ((ka,kb,kc); (k e 0)) 替换 ((a,b,c)) 所得 (f) 值不变),不妨设 (x+y+z=1),则

    [egin{aligned} f&= dfrac{(x+1)^2}{2x^2+(x-1)^2}+dfrac{(y+1)^2}{2y^2+(y-1)^2}+dfrac{(z+1)^2}{2z^2+(z-1)^2} \ &= dfrac 13 left[1+dfrac{8x+2}{3(x-frac 13)^2+frac 23}+1+dfrac{8y+2}{3(y-frac 13 )^2+frac 23}+1+dfrac{8z+2}{3(z-frac 13)^2+frac 23} ight] \ &leq dfrac 13 left[3 + dfrac 32 left(8x+8y+8z+6 ight) ight] = 8 end{aligned} ]

    等号取到当且仅当 (x=y=z= dfrac 13),满足条件,所以 (f_{max} = 8).

    Problem 8

    (x,y,z > 0)(xyz=1),求证:

    [dfrac{x^5-x^2}{x^5+y^2+z^2}+dfrac{y^5-y^2}{x^2+y^5+z^2}+dfrac{z^5-z^2}{x^2+y^2+z^5} geq 0 ]


    Analysis

    把分子上的五次项和分母配凑拿掉之后,构造 ( m Cauchy) 不等式,由于轮换对称到最后的分子会和分母统统约掉,完了这里边水题越来越多了(

    我的洛星玖啊,因为没法边穿可爱的小裙子边抑制自己做水题的冲动,气的内心崩溃,又哭又闹,呜呜呜呜,好可怜啊(((

    Solution

    易证,原不等式等价于

    [dfrac {x^2+y^2+z^2}{x^5+y^2+z^2} + dfrac {x^2+y^2+z^2}{x^2+y^5+z^2} + dfrac {x^2+y^2+z^2}{x^2+y^2+z^5} leq 3 ]

    那么考虑利用条件 (xyz=1) 构造如下 ( m Cauchy-Schwarz) 不等式,其中五次项关于 (x,y,z) 对称.

    [(x^5+y^2+z^2)(yz+y^2+z^2) geq left(x^2+y^2+z^2 ight)^2 ]

    由此,原不等式转化为

    [egin{aligned} { m LHS} &leq dfrac{(yz+y^2+z^2)+(xz+x^2+z^2)+(xy+x^2+y^2)}{x^2+y^2+z^2} \ &leq dfrac{2(x^2+y^2+z^2)+x^2+y^2+z^2}{x^2+y^2+z^2} = 3 quadquad { m applying; Cauchy-Schwarz; Inequality} end{aligned} ]

    Problem 9

    已知对任意 (x_1, x_2, cdots x_{2016} in [0,4]) 方程

    [sum_{i=1}^{2016} |x-x_i| = 2016k ]

    ([0,4]) 上至少有一个根,试证明实数 (k) 唯一并求其值.


    Analysis

    总算没那么水了,实际上这是个嵌套最值问题,考虑从不等式角度来思考原方程有解的条件,就能得到 (max min f(x) le 2016k le min max f(x))

    然后需要利用绝对值和函数的一些小 ( m trick) 就能发现两边是相等的,随之也就解出来 (k) 了.

    Solution

    (f(x) = sum |x-x_i|),由于 (f(x) = 2016k)([0,4]) 上有解,那么 (min f(x) le 2016k le max f(x)).

    (f(x))(x_1, x_2, cdots x_{2016}) 变化而变化,所以对于每一组 ((x_1,x_2,cdots,x_{2016})) 都有上述不等式成立,等价于 (max min f(x) le 2016k le min max f(x)).

    由于 (x_i) 的任意性,不妨设 (x_1 le x_2 le cdots le x_{2016}),那么根据绝对值和函数的性质,

    [egin{aligned} max f(x)_{min} &= max f(x) igg|_{x in [x_{1008},x_{1009}]} \ &= max_{(x_1,x_2,cdots,x_{2016})} sum_{i=1}^{1008} (x-x_i) + sum_{j=1009}^{2016} (x_j-x) \ &= max_{(x_1,x_2,cdots,x_{2016})} sum_{i=1009}^{2016} x_j - sum_{i=1}^{1008} x_i \ &xlongequal[]{{ m Substitute;}(0,cdots,4)} 4 imes 1008 = 2 imes 2016 end{aligned} ]

    再求不等式右边,继续利用绝对值和函数在最小、最大间断点、左边界或右边界上取最大值的性质,我们得到

    [egin{aligned} min f(x)_{max} &= min f(x) igg|_{x in {0,4}} \ &= min_{(x_1,x_2,cdots,x_{2016})} left{sum_{i=1}^{2016} x_i, 4 imes 2016 - sum_{i=1}^{2016} x_i ight} \ &geq min_{(x_1,x_2,cdots,x_{2016})} 2 imes 2016 = 2 imes 2016 end{aligned} ]

    所以我们有 (2 imes 2016 leq 2016k leq 2 imes 2016),得到 (k = 2).

    Problem 10

    (f(x)=x^2+a),记 (f^1(x)=f(x), f^{(n)}(x)=f(x) circ f^{(n-1)}(x) ; (n = 2,3, cdots)),求对于所有正整数 (n) 满足 (left| f^{(n)}(0) ight| le 2) 的所有实数 (a) 构成的集合 (S).


    Analysis

    嗯,目前最有难度的一道题,说实话有点分析的意思. 首先我么可以把范围缩小到 ([-2,2]) 上.

    原题实际上是给出了一个无穷数列 ({|f(0)|, left|f^{(2)}(0) ight|, cdots}),我们需要探究它在什么条件下是收敛的,而且极限值还要小于等于 (2).

    当然我们可以令 (g(n)=f^n(0)),直接研究 (g) 是很困难的,那么我们想到 ( m Stolz) 定理,就去研究 ({ m d}g),当然离散到数列上叫 (Delta g).

    (Delta g) 会被配凑出来二次函数形式,这样我们再通过求和就能给出一个数列收敛的上界,这时我们发现无事可做了,那么利用归纳法尝试证明该上界为上确界,此时该题就被解决了。

    Solution

    (f(0) leq 2) 得到,(|a| leq 2),即 (-2 leq a leq 2),所以 (S subseteq [-2,2]).

    由题 (forall ; i in N^*, f^{(i+1)}(0)=f(f^{(i)}(0))),所以

    [egin{aligned} f^{(i+1)}(0) - f^{(i)}(0) &= f(f^{(i)}(0)) - f^{(i)}(0) \ &= left[f^{(i)}(0) ight]^2+a-f^{(i)}(0) \ &= left[f^{(i)}(0) - frac 12 ight]^2 + a - dfrac 14 geq a - dfrac 14 end{aligned} ]

    所以当 (a ge dfrac 14)

    [egin{aligned} f^{(n)}(0) &= sum_{i=1}^{n-1} left[f^{(i+1)}(0) - f^{(i)}(0) ight] + f(0) \ &geq (n-1)left(a-dfrac 14 ight)+a end{aligned} ]

    由于 (a-dfrac 14>0),所以该式随 (n) 的递增而递增,也就是说,当 (n > dfrac{2-a}{a-frac 14}+1) 时,(|f^{(n)}(0)|>2),所以 (S subseteq left[-2,dfrac 14 ight]).

    下证 (S = left[-2, dfrac 14 ight]),先证 (left[0,dfrac 14 ight] subseteq S),尝试利用归纳法证明加强命题 (f^{(i)}(0) leq dfrac 12).

    首先 (f(0) = a leq dfrac 12)(f^{(2)}(0) = a^2+a leq dfrac 12),那么归纳的假设 (f^{(i)}(0) leq dfrac 12),则 (f^{(i+1)}(0)=left[f^{(i)}(0) ight]^2+a leq dfrac 12),得证.

    再证 ([-2, 0] subseteq S),尝试利用归纳法证明 (-a leq f^{(i)}(0) leq a).

    首先 (|f(0)| = -a leq 2)(left|f^{(2)}(0) ight| = |a^2+a| leq |-2a+a| = |a|),那么归纳的假设 (left|f^{(i)}(0) ight| leq -a),则 (left|f^{(i+1)}(0) ight| = left| left[f^{(i)}(0) ight]^2+a ight| leq |a^2+a| leq |a|),得证.

    综上 ([-2,0] cup left[0, dfrac 14 ight] subseteq S)(S subseteq left[-2, dfrac 14 ight]),所以 (S = left[-2, dfrac 14 ight]).

    Problem 11

    (f{IMO\,2020;/;Problem\,2})

    任取正数 (a ge b ge c ge d ge 0),满足 (a+b+c+d=1),求证:

    [(a+2b+3c+4d)a^ab^bc^cd^d < 1 ]


    Analysis

    这题真的是 (f{IMO}) 题哎,不过好像确实挺新颖的,考虑利用加权均值不等式把很麻烦的 (a^ab^bc^cd^d) 转化成 (a^2+b^2+c^2+d^2).

    发现左边是三次,那么我们尝试把 (1) 换成 ((a+b+c+d)^3),用多项式乘法打开就证完了。

    Solution

    Problem 12

    给定整数 (n ge 3),实数列 ({x_n}) 满足 (min limits_{1 le i < j le n} left|x_i-x_j ight| = 1, f = sum limits_{i=1}^n |x_i|),求 (f_{min}).


    Analysis

    Solution

    Problem 13

    (f {USAMO \, 2020 ; / ; Problem \,6})

    给定正整数 (n ge 2). 若实数列 ({x_n}, {y_n}) 满足 (x_1 ge x_2 ge cdots ge x_n,y_1 ge y_2 ge cdots ge y_n),且

    [sum_{i=1}^n x_i = sum_{i=1}^n y_i = 0, sum_{i=1}^n x_i^2 = sum_{i=1}^n y_i^2 = 1 ]

    求证

    [sum_{i=1}^n (x_iy_i - x_iy_{n-i+1}) geq dfrac{2}{sqrt{n-1}} ]


  • 相关阅读:
    (转)EDM邮件制作规范完整版
    (转)Gmail,你必须了解的12个邮件编码问题
    说说CakePHP的关联模型之一 基本关联
    HTML5 离线应用程序
    CakePHP模型中使用join的多种写法
    判断浏览器
    Javascript闭包例子
    安装wamp后,其显示目录的图标显示不出来
    underscore.js 分析 第二天
    HTML5心得
  • 原文地址:https://www.cnblogs.com/herself32-lyoi/p/14092285.html
Copyright © 2020-2023  润新知